OMC214


Online Math Contest

대회 링크
https://onlinemathcontest.com/contests/omc214

A

문제

평면 위에 정사각형이 있고, 꼭짓점과 함께 각 변을 5등분하는 점들을 합쳐 20개의 점들을 재밌는 점이라고 부르겠습니다.
정확히 2개의 재밌는 점을 지나는 평면 위의 직선은 몇 개 있을까요?

해설

\(3\)개 이상의 재밌는 점들이 일직선상에 있을 때, 이 직선은 정사각형의 변 중 하나를 연장한 것과 일치합니다.
따라서, 재밌는 점들 중 \(2\)가지를 고르는 \(_{20}C_2 = 190\)개의 경우 중에서 정사각형의 같은 변 위에 있는 \(2\)개의 점을 지나는 경우인 \(4 \times _6C_2 = 60\)개를 빤 \(130\)개는 정확히 \(2\)개의 재밌는 점을 지나는 서로 다른 직선의 수와 대응합니다.
따라서 구하는 답은 \(130\)입니다.

B

문제

수평면과 그것에 대해 기울어진 평면이 있습니다. 이 평면의 서쪽 방향으로의 기울기는 \(- \frac{1}{3}\), 남쪽 방향으로의 기울기는 \(- \frac{1}{4}\)입니다.
이 떄, 이 평면의 임의의 방향에 대해서의 기울기가 실수 \(s\)이하라고 하면, \(s\)의 최솟값을 서로소인 두 양의 정수 \(a, b\)를 이용홰 \(\frac{a}{b}\)와 같이 나타낼 수 있습니다.
\(a + b\)를 구하세요.


여기서 수평면에서의 어떤 방향에 대한 기울기라 함은, 그 방향에서 수평으로 \(1\)만큼 나아갔을 때의 높이의 변화를 가리킵니다.

해설

이 평면은 \(A(0, 0, 0,), X(3, 0, 1), Y(0, 4, 1)\)을 지나는 평면으로 봐도 무방합니다. 또한, 공간 안의 점 \(P\)를 \(xy\)평면 위에 직교 투영한 점을 \(P’\)라고 합시다.
직선 \(XY\)위의 임의의 점 \(P\)에 대해, \(PP’ = 1\)이라고 하면 \(AP\)방향의 기울기는 \(\frac{1}{AP’}\)가 됩니다. 또한, \(P\)를 임의로 움직이면 \(P’\)는 직선 \(X’Y’\)위를 움직이게 되므로, 구하는 답은 \(A\)와 \(X’Y’\)의 거리의 역수가 됩니다.
직선 \(X’Y’\)는 \(4x + 3y -12 = 0\)으로 나타낼 수 있으므로,

\[s = \frac{\sqrt{4^2 + 3^2}}{\vert 4 \times 0 + 3 \times 0 - 12 \vert} = \frac{5}{12}\]

가 됩니다. 따라서 구하는 답은 \(17\)입니다.

C

문제

\(1\)부터 \(9\)까지의 정수를 \(1\)개씩 원형으로 나열하는 방법 중에서, 이웃한 어떤 \(2\)개의 수도 서로소가 되도록 하는 방법은 몇 개나 있을까요? 단, 회전해서 일치하는 경우는 같은 것으로 처리하며, 뒤집어서 일치하는 것은 다른 것으로 처리합니다.

해설

\(1\)부터 \(9\)까지의 정수를 다음과 같이 분류합시다. 같은 그룹에 속하는 것은 묶어서 생각하면 됩니다:

\[A: \{1, 5, 7\}, B: \{2, 4, 8\}, C: \{3, 9\}, D: \{6\}\]

이렇게 분류했을 때, 이웃해서는 안 되는 수인 경우는 \(BB, CC, BC, DC, DD\)일 때 입니다. 특히, \(D\)의 양 옆은 \(A\)여야만 합니다.
\((A, D, A)\)를 고정했을 때, 남은 \(A, B, B, C, C\)를 나열하는 방법은 다음과 같은 \(10\)개가 있습니다.

\(D\)의 위치를 고정하고, 각 그룹에 대한 수의 분배를 생각해보면, 구하는 답은 \(10 \times 3! \times 3! \times 2! \times 1! = 720\)개가 됩니다.

D

문제

실수 \(a, b, c, d, e\)가 다음의 식을 만족할 때, \(e\)를 구하세요.

\[\begin{cases} 9^4a + 11 ^4b + 19^4c + 21^4d = -1 + e \\ 19^4a + 21^4b + 39^4c + 41^4d = -1 \\ 29^4a + 31^4b + 59^4c + 61^4d = -1 \\ 39^4a + 41^b + 79^4c + 81^4d = -1 \\ 49^4a + 51^4b + 99^4c + 101^4d = -1\end{cases}\]

단, \(e\)는 서로소인 두 양의 정수 \(p, q\)를 이용해 \(e = \frac{p}{q}\)로 나타낼 수 있습니다.
\(p + q\)를 제출해 주세요.

해설

\(f(x) = x^4 + a(x - 1)^4 + b(x + 1)^4 + c(x - 2)^4 + d(x + 2)^4\)라고 두면 문제에 제시된 아래쪽 \(4\)개의 식으로부터

\[f\left(\frac{1}{20}\right) = f\left(\frac{1}{30}\right) = f\left(\frac{1}{40}\right) = f\left(\frac{1}{50}\right) = 0\]

입니다. \(f(x)\)는 \(4\)차식이므로, 어떤 실수 \(r\)에 대해 다음의 항등식이 성립합니다.

\[f(x) = r\left(x - \frac{1}{20}\right)\left(x - \frac{1}{30}\right)\left(x - \frac{1}{40}\right)\left(x - \frac{1}{50}\right)\]

양변의 \(4\)차항의 계수, \(2\)차항의 계수, 정수항을 비교해 다음 3개의 식을 얻을 수 있습니다.

\[\begin{cases} 1 + (a + b) + (c + d) = r \\ 6(a + b) + 24(c + d) = \frac{71r}{12 \times 10^3} \\ (a + b) + 16(c + d) = \frac{r}{12 \times 10^5} \end{cases}\]

이를 풀면 \(r = \frac{288 \times 10^5}{288 \times 10^5 - 355 \times 10^2 + 6}\)이 됩니다. 따라서,

\[e = 10^4f\left(\frac{1}{10}\right) = \frac{2880000}{14382253}\]

입니다. 따라서 구하는 답은 \(17262253\)입니다.

E

문제

\(1\)이상 \(5000\)이하의 정수 중 \(1\)개가 쓰인 \(1 \times 2\)의 타일이 \(5000\)개 있고, 어떤 \(2\)개의 타일에도 같은 수가 쓰여있지 않습니다.
이 타일들을 겹치거나 하지 않고 \(100 \tiems 100\)칸을 빈틈없이 모두 채운다고 할 떄, 다음 조건을 만족했습니다.

또한, 위에서 \(a\)행쨰, 왼쪽에서 \(b\)열째의 칸을 덮는 타일에 쓰인 수를 \(N_{a, b}\)라고 합시다. 이 때,

\[N_{1, 51} + N_{2, 50} + N_{3, 31} + N_{4, 50} + … + N_{99, 51} + N_{100, 50}\]

으로 가능한 최댓값을 구하세요.

해설

\(n\)을 임의의 양의 정수라 두고 \(100\)을 \(2n\)이라고 바꿔서 생각해봅시다.
위에서 \(a\)행째, 왼쪽에서 \(b\)열째의 칸을 \((a, b)\)라 합시다. 또,

\[(1, n + 1), (2, n), (3, n + 1), (4, n), …, (2n - 1, n + 1), (2n, n)\]

을 [중앙의 칸]이라 하며, 중앙의 칸으로부터 왼쪽에 있는 칸을 간단히 [왼쪽의 칸], 중앙의 칸으로부터 오른쪽에 있는 칸을 간단히 [오른쪽의 칸]이라고 합시다. 또, \(a + b\)가 짝수라면 il \((a, b)를\) 흰색으로 칠하고, 홀수라면 검은색으로 칠합니다.

왼쪽의 칸들 중 \((1, nj)\)과 같은 색의 칸은 \(n^2\)개, 다른 색의 칸은 \(n^2 - n\)개 있습니다. 어떤 타일도 검은 칸과 흰 칸을 하나씩 덮는 것으로부터, 왼쪽 칸들만 덮고있는 타일은 많아야 \(n^2 - n\)개입니다. 마찬가지로, 오른쪽 칸을 덮는 타일도 많아야 \(n^2 - n\)개입니다.
거기에, 왼쪽 칸과 오른쪽 칸을 동시에 덮는 타일은 존재하지 않으므로, 중앙의 칸이면서 \(n^2 - n + 1\)이하인 정수가 쓰인 타일에 덮인 칸이 존재합니다. 따라서,

\[N_{1, n + 1} + N_{2, n} + … + N_{2n - 1, n + 1}, N_{2n, n} \leq n^2 - n + 1 + \sum_{k = 0}^{2n - 2}(2n^2 - k) = n(4n^2 - 3n + 2)\]

입니다.

등식이 성립하는 타일의 배치가 존재하므로, \(n = 50\)인 경우에 구하는 답은 \(492600\)입니다.

F

문제

반지름이 \(169\)인 원에 내접하는 삼각형 \(ABC\)에 대해, 변 \(AB, AC\) 위에 각각 점 \(D, E\)가 있으며, 삼각형 \(ABC, ADE\) 각각의 외접원의 점 \(A\)가 아닌 교점을 \(F\)라 할 때, 다음이 성립했습니다:

\[DF = EF = 130, DE = 100\]

여기에, 선분 \(BE\)와 선분 \(CD\)는 점 \(X\)에서 직교했습니다. 선분 \(BC, DE\)의 중점을 각각 \(M, N\)이라 했을 때, 서로소인 두 양의 정수 \(a, b\)를 이용해 \(\cos\angle{MXN} = -\frac{a}{b}\)로 나타낼 수 있습니다. 이 때 \(a + b\)의 값을 구하세요.

해설

\(F\)는 사각형 \(BCED\)에 대한 미켈 점인 것으로부터, 삼각형 \(FBC\)와 \(FDE\), 삼각형 \(FBD\)와 \(FCE\)와 \(FMN\)은 닮음입니다.
\(DF = EF\)인 것으로부터, 삼각형 \(FBD\)와 \(FCE\)는 합동이며, 삼각형 \(FBC\)는 이등변삼각형입니다. 따라서 \(BD = CE\)이며, 이 길이를 \(x\)라 두면 \(BE \perp CD\)인 것으로부터 다음이 성립합니다.

\[2x^2 = BD^2 + CE^2 = BC^2 + DE^2\]

또,

\[BD : MN = DF : FN = 13 : 12\]

입니다. 거기에, 삼각형 \(ABC\)의 외심을 \(O\), 선분 \(BF\)의 중점을 \(K\)라 두면, 삼각형 \(FOK\)와 \(FBM\)은 닮음이므로,

\[BC = 2BM = 2 \times KO \times \frac{BF}{FO} = 4 \times \frac{5}{13} FO \times \frac{FK}{FO} = \frac{240}{169} FO = 240\]

인 것으로부터, 코사인 법칙(여현정리)에 의해 다음과 같이 답을 구할 수 있습니다.

\[\begin{align} \cos \angle{MXN} & = \frac{XM^2 + XN^2 - MN^2}{2 \times XM \times XN} \nonumber\\ & = \frac{BC^2 + DE^2 - 4 \times \left(\frac{12}{13}x\right)^2}{2 \times BC \times DE} \nonumber\\ & = -\left(2 \times \left(\frac{12}{13}\right)^2 - 1 \right) \times \frac{BC^2 + DE^2}{2 \times BC \times DE} \nonumber\\ & = -\frac{119}{120} \nonumber \end{align}\]

따라서 구하는 답은 \(239\)입니다.

A, C는 풀었다

역시 수학은 그렇게 자신있는 분야는 아니라, 이번엔 A와 C 두 문제를 푸는 것에 그쳤다.
B 같은 경우 조금 더 생각했으면 풀었을 수도 있겠지만, 시간 내엔 무리였다.

더 실력을 쌓아야지.

© 2024 SeokguKim   •  Base Theme  Moonwalk